A parallelogram is a quadrilateral with two pairs of parallel sides.

A quadrilateral is a parallelogram _[blank]_.

Which answer correctly fills in the blank in the previous sentence?

if it has only two pairs of parallel sides
if and only if it has two pairs of parallel sides
because it has only four sides and two of them are parallel
if it has two pairs of parallel sides and is a polygon

Answers

Answer 1

The quadrilateral is a parallelogram if and only if it has two pairs of parallel sides

What is parallelogram?

In two-dimensional geometry, it is a plane shape having four sides, in which two pairs of sides are parallel to each other and equal in length. The sum of all angles in a parallelogram is 360°.

It is given that:

A parallelogram is a quadrilateral with two pairs of parallel sides.

As we know,

The quadrilateral can be defined as the four-sided polygon in geometry having four edges and four corners.

The quadrilateral, has:

Two pairs of congruent sides.

It has one pair of opposite congruent angles.

The diagonals of a kite are perpendicular.

One diagonal is bisected.

The top and bottom angles are bisected but does not have congruent diagonals.

A quadrilateral is a parallelogram if and only if it has two pairs of parallel sides.

To Learn more about the parallelogram here:

brainly.com/question/1563728

#SPJ1


Related Questions

A cement mix calls for 2 cups of water and 1 1/4 pounds of mix to make five large tiles stick to the ground, Yolanda has plenty of water but only 4 3/8 pounds of mix. How many large tiles (including parts of a tile) can she stick to the ground? You MUST think conceptually and illustrate your thinking with a model. Do NOT use proportional reasoning.

Answers

175/26 large tiles can she stick to the ground.

What is Equation?

Two or more expressions with an Equal sign is called as Equation.

Given,

A cement mix calls for 2 cups of water and [tex]1\frac{1}{4}[/tex] pounds of mix to make five large tiles stick to the ground,

2+ [tex]1\frac{1}{4}[/tex]=5

2+5/4

13/4

Which means 13/4 is needed for 5 large tiles.

Yolanda has plenty of water but only [tex]4\frac{3}{8}[/tex] pounds of mix

We need to find how many large tiles can she stick to the ground.

Let x be the number of tiles.

[tex]4\frac{3}{8}[/tex] =35/8

So

(13/4)/5=(35/8)/x

Apply cross multiplication

13/4x=35/8×5

13/4x=175/8

x=175/8×4/13

x=700/104=350/52=175/26

Hence, 175/26 large tiles can she stick to the ground.

To learn more on Equation:

https://brainly.com/question/10413253

#SPJ1

NO LINKS!!

f(x)= (x^2 + x - 12)/(x^2 + 6x + 9)

Discuss the behavior of f near any excluded x-values/
a. f(x) --> -∞ as x --> 3^+ and as x--> 4^-, f(x) --> ∞ as x --> 4^+ and as x --> 3^-
b. f(x) --> ∞ as x --> 4^+ and as x --> 4^-
c. f(x) --> -∞ as 4^+ and as x --> 4^-, f(x) --> ∞ as 3^+ and as x --> 3^-
d. f(x) --> -∞ as x --> 3^+ and as x --> 3^-, f(x) --> ∞ as x --> -4^+ and as x --> -4^-
e. f(x) --> -∞ as x --> -3^+ and as x --> -3^-

Answers

Answer:

  e. f(x) --> -∞ as x --> -3^+ and as x --> -3^-

Step-by-step explanation:

You want to know the behavior of f(x)= (x^2 + x - 12)/(x^2 + 6x + 9) near any excluded x-values.

Domain

The function can be factored as ...

  [tex]f(x)=\dfrac{x^2+x-12}{x^2+6x+9}=\dfrac{(x+4)(x-3)}{(x+3)^2}[/tex]

The excluded values are values of x where the denominator is zero. The only excluded value is x = -3. (eliminates all answer choices except E)

Asymptotic behavior

At either side of x = -3, the sign of the numerator is negative and the sign of the denominator is positive. That makes f(3-) < 0 and f(3+) < 0.

f(x) will never approach +∞, but f(x) approaches -∞ as x nears -3 from either direction.

Answer:

[tex]\textsf{e)} \quad f(x) \rightarrow -\infty\;\;\textsf{as}\;\; x \rightarrow -3^+ \;\;\textsf{and as}\;\;x \rightarrow -3^-[/tex]

Step-by-step explanation:

Given function:

[tex]f(x)=\dfrac{x^2+x-12}{x^2+6x+9}[/tex]

Factor the numerator:

[tex]\implies x^2+x-12[/tex]

[tex]\implies x^2+4x-3x-12[/tex]

[tex]\implies x(x+4)-3(x+4)[/tex]

[tex]\implies (x-3)(x+4)[/tex]

Factor the denominator:

[tex]\implies x^2+6x+9[/tex]

[tex]\implies x^2+3x+3x+9[/tex]

[tex]\implies x(x+3)+3(x+3)[/tex]

[tex]\implies (x+3)(x+3)[/tex]

[tex]\implies (x+3)^2[/tex]

Therefore, the rational function is:

[tex]f(x)=\dfrac{(x-3)(x+4)}{(x+3)^2}[/tex]

As the degree of the numerator is equal to the degree of the denominator, there is a horizontal asymptote at y = 1.

A vertical asymptote occurs at the x-value(s) that make the denominator of a rational function zero.

[tex]\implies (x+3)^2=0[/tex]

[tex]\implies x+3=0[/tex]

[tex]\implies x=-3[/tex]

Therefore, there is a vertical asymptote at x = -3.

As there is a vertical asymptote at x = -3, the excluded x-value is x = -3.

As x approaches x = -3 from both sides, the numerator of the rational function approaches -6 and the denominator approaches a very small positive number.  Therefore, the function approaches a very large negative number.

Therefore, the end behaviour of the function as it approaches the excluded value is:

[tex]f(x) \rightarrow -\infty\;\;\textsf{as}\;\; x \rightarrow -3^+ \;\;\textsf{and as}\;\;x \rightarrow -3^-[/tex]

A political action committee sends out a questionnaire to randomly-selected mailing addresses, asking people to rate the importance of a variety of economic and social issues facing the country. At the end of the questionnaire is an (optional) invitation to donate money to the organization. Which of the following statements about this survey is true?
A) The survey results are invalid because it’s impossible to word questions about politics without creating bias.

B) The survey results are invalid because only surveys conducted by phone are free from bias.

C) The survey results will underestimate support for this political action committee in the entire population because people who respond are less likely to support an organization that asks for money.

D) The survey results will overestimate support for this political action committee in the entire population because people more likely to respond if they are prepared to donate money.

E) Since the survey was sent to randomly-selected households, it should be free of bias.

Answers

Option c; The survey results will underestimate support for this political action committee in the entire population because people who respond are less likely to support an organization that asks for money. This is the correct statement about the survey.

What is politics?

Politics is a category of activities that are related to group decision-making or other types of power dynamics between people, such as the allocation of resources or status. Political science is the name of the area of social science that focuses on politics and governance.

Given,

A political action committee sends out a questionnaire to randomly-selected mailing addresses, asking people to rate the importance of a variety of economic and social issues facing the country. at the end of the questionnaire is an (optional) invitation to donate money to the political action committee.

We have to find the correct statement about the survey from the given options;

Because respondents are less inclined to support a group that solicits donations, the survey results will understate public support for this political action committee.

The correct statement about the survey is option c;

Learn more about political action committee here;

brainly.com/question/26270345

#SPJ1

DUE SOON!!! NEED HELP ON THESE TWO QUESTIONS PLEASE!
DUE SOON AND I DONT UNDERSTAND IT!
MANY THANKS!
WILL GET 100 POINTS! PLEASE HELP!!

Answers

The restrictions for x and y are given as follows:

3. x ∈ W, y ∈ W.

5. x ∈ W, y ∈ W.

What are the restrictions?

To obtain the restrictions, the first step is classifying the variables as either discrete or continuous, as follows:

Discrete variables are countable values, that can assume only whole or integer values, that is, they do not assume decimal values.Continuous variables can assume decimal values, hence they are real values.

In this problem, the variables are both resumed as follows:

Number of tickets sold in the Super Bowl.

The possible values are given as follows:

0, 1, 2, ...

Which are countable values, hence discrete, and that also cannot assume negative values, hence they belong to the whole set and not the integer set, thus the restrictions are:

x ∈ W, y ∈ W.

More can be learned about discrete and continuous variables at https://brainly.com/question/3130222

#SPJ1

Let P (t) represent the amount of _ certain drq milligrams; that = he bloodstream time hou? The rate at which the= drug leaves the bloodstream proportional to the amount of the drug bloodstream: The amount of the drug in the bloodstream can be modeled by # KP ; where constant and- time; hours At time 500 milligrams of the drug was administered. At the moment when the amount of the drug in the bloodstream 50 milligrams, the amount decreasing at a rate milligrams per hour Which of the following an expression for P (t) P (t) = 500 0.23t P (t) = 500 4t P (t) = s0e 0.23 P (t) = e-02x + 499

Answers

As proportional relationship, the amount decreasing at a rate milligrams per hour expression P(t) = 500 + e⁻⁰²ˣ

Proportional relationship:

In math, the relationships between two variables where their ratios are equivalent is known as proportional relationship.

Given,

Here we have the rate at which the= drug leaves the bloodstream proportional to the amount of the drug bloodstream: The amount of the drug in the bloodstream can be modeled by K; where constant and- time; hours At time 500 milligrams of the drug was administered. At the moment when the amount of the drug in the bloodstream 50 milligrams, the amount decreasing at a rate milligrams per hour.

Here we need to find the expression.

While we looking into the given question, we have identified the following details,

=> Bloodstream = 500 milligrams

Here they state that the bloodstream 50 milligrams, the amount decreasing at a rate milligrams per hour.

So, the expression for this proportional relationship is written as,

=> P(t) = 500 + e⁻⁰²ˣ

To know more about Proportional relationship here.

https://brainly.com/question/29765554

#SPJ4

S is the midpoint of RT. T has coordinates (-14,-15) and S has coordinates (11,-1) Find the coordinates of R

Answers

Answer:

(36, 13)

Step-by-step explanation:

to find the midpoint you have the add the x and the y and then divide by two

to find one of the endpoints you just do the opposite

you multiply the midpoint by 2

and then subtract the remaining endpoint from it

(11, -1) --> (22, -2)

now subtract T from it

(36, 13)

that is your answer

Find the Maclaurin series for f(x) = xe^2x and its radius of convergence. Use only the definition of a Maclaurin series.

Answers

The Maclaurin Series of xe²ˣ: x + 2x² + 2x³+ 4/3x⁴+ 2/3x⁵ +....

Given that,

f(x) = xe^2x

Taylor series of function f(x) at a is defined as:

f(x) = f(a) + f'(a)/1!/(x-a) +f''(a)/2! (x-a)² +....

Maclaurin series of Function F(x) is a Taylor series of Function f(x) at: a=0

f(x) = f(0) + f'(0)/1! /(x) +f''(0)/2!/ (x)² +....

Apply the Maclaurin Formula

= 0 +d/dx (xe²ˣ)(0)/1! x + d²/dx² (xe²ˣ)(0)/2! x² +....

Evaluate the derivatives :

= 0 + 1/1! x + 4/2! x² + 12/3! x³ + ...

Refine :

= x + 2x² + 2x³+ 4/3x⁴+ 2/3x⁵ +....

So, the Maclaurin series for f(x) = xe^2x is  x + 2x² + 2x³+ 4/3x⁴+ 2/3x⁵ +....

To learn more about Maclaurin Series click here:

brainly.com/question/24188694

#SPJ4

Jones Co. borrowed money that is to be repaid in 12 years. So that the loan will be paid back at end of the 12th year, the company invests $8,000 at end of each year at 5% compounded annually. The amount of the original loan was
Multiple Choice
$127,337.01
$70,905.60
$127,636.80
$70,950.60

Answers

The required future value of the annuity is $127,336.80.

Jones Company borrowed money that must be paid back in 12 years. To ensure that the debt is repaid by the end of the 12th year, the corporation invests $8,000 at the end of each year at a rate of 5% compounded yearly.

Compute the future value of the annuity as follows:

Future value = Annuity - FVIFA

FVIFA is taken from the table, and substitutes the value of the annuity in the above formula

Future value = $8,000 × 15.9171

Apply the multiplication operation, and we get

Future value = $127,336.80

Therefore, the required future value of the annuity is $127,336.80.

To learn more about the annuity click here:

brainly.com/question/13247656

#SPJ1

The stock of WAL-MAT pays a dividend of $.88. The stock opened at $18.25 and closed at $18.33. The stock yield is:
Multiple Choice
4.8%
4.2%
4.1%

Answers

The stock yield for the stocks will be 4.8%. The correct option is A.

What is an expression?

The mathematical expression combines numerical variables and operations denoted by addition, subtraction, multiplication, and division signs.

Mathematical symbols can be used to represent numbers (constants), variables, operations, functions, brackets, punctuation, and grouping. They can also denote the logical syntax's operation order and other properties.

To get the stock to yield use this formula - annual dividend / current stock price or share price

Substitute the dividend of $.88 and the closing price of $18.33 to the formula above. Divide and simplify.

Stock yield = .88 / 18.33

Stock yield = 0.0480 x 100

Stock yield = 4.8%

Therefore, the stock yield for the stocks will be 4.8%. The correct option is A.

To know more about an expression follow

https://brainly.com/question/15055460

#SPJ1

In Norway last year, the lowest temperature was −15°C. In Norway last year, the highest temperature was 42°C greater than the lowest temperature. Work out the highest temperature in Norway last year

Answers

Answer:

27°

Step-by-step explanation:

Let h = highest temperature

h = -15 + 42

h = 27°

A travel company has hired a management consultant company to analyze demand in twenty-six regional markets for one of its major products: a guided tour to a particular country. The consultant uses data to estimate the following equation:
Q=1500−4P+5A+10I+3PY
Where Q = amount of the product demanded;
P = price of the product in dollars;
A = advertising expenditures in thousands of dollars;
I = income in thousands of dollars;
PY = price of some other travel products offered by a competing travel company.
a) Calculate the amount demanded for this product using the following data:
P = $400
A = $20,000
I = $15,000
PY=$500
b) Suppose the competitor reduced the price of its travel product to $400 to match the price of this firm's product. How much would this firm have to increase its advertising in order to counteract the drop in its competitor's price? Would it be worth for them to do so? Explain your answer.
c) What other variables might be important in helping estimate the demand for this travel product?

Answers

Q falls to 251,400 to 201,400 which means a fall of 50,000 which should be counteracted by advertising. If advertising expenditure becomes 30,000 it will raise the Q again to 251,400.

Given that,

One of a travel company's main products, a guided tour to a specific country, was the subject of a demand analysis by a management consulting firm for 26 regional markets. The consultant estimates the following equation using data:

Q=1500−4P+5A+10I+3PY

Q is the quantity of the product that is demanded;

P represents the item's monetary price;

A represents advertising costs in thousands of dollars;

I represents income expressed in dollars;

PY is the cost of a different travel product provided by a rival travel agency.

We have to find

(a) Utilizing the information below, determine the amount demanded for this product:

P = $400

A = $20,000

I = $15,000

PY=$500

Q = 1,500 - 4 × 400 + 5 × 20,000 + 10 × 15,000 + 3 × 500

= 1,500 - 1,600 + 100,000 + 150,000 + 1,500

= 251,400

(b)  In Income falls to 10,000

Q = 1,500 - 4 × 400 + 5 × 20,000 + 10 × 10,000 + 3 × 500

= 1,500 - 1,600 + 100,000 + 100,000 + 1,500

= 201,400

Q falls to 251,400 to 201,400 which means a fall of 50,000 which should be counteracted by advertising. If advertising expenditure becomes 30,000 it will raise the Q again to 251,400.


To learn more about advertising visit: https://brainly.com/question/17761623

#SPJ4

where is root 19??????

Answers

Answer:

4.358

Step-by-step explanation:

sqrt(19) = 4.358

The price of a gift plus 14% delivery charge comes to a total cost of $19.38. What was the price of the gift?

Step 1 of 2 : Describe the above situation as a linear equation, using "x" or "y" as variable names to describe the unknown quantity.

Answers

Answer:

1.14x = 19.38

$17

Step-by-step explanation:

Let the price before delivery charge = x.

The charge is 14%, so it is 14% of x, or 0.14x.

The original price plus charge is x + 0.14x, or 1.14x.

1.14x = 19.38

x = 19.38/1.14

x = 17

The price is $17

sin(theta) * sec(theta) - 2sin(theta) = 0

Solve with all possible answers for theta
(please)

Answers

Answer:

θ = 2 π n_1 + π/3 for n_1 element Z

or θ = 2 π n_2 + (5 π)/3 for n_2 element Z

or θ = π n_3 for n_3 element Z

θ = π n_3 for θ element Z and cos(θ)!=0 and n_3 element Z

Step-by-step explanation:

Solve for θ:

tan(θ) - 2 sin(θ) = 0

Factor sin(θ) from the left hand side:

sin(θ) (sec(θ) - 2) = 0

Split sin(θ) (sec(θ) - 2) into separate parts with additional assumptions.

Assume cos(θ)!=0 from sec(θ):

sec(θ) - 2 = 0 or sin(θ) = 0 for cos(θ)!=0

Add 2 to both sides:

sec(θ) = 2 or sin(θ) = 0 for cos(θ)!=0

Take the reciprocal of both sides:

cos(θ) = 1/2 or sin(θ) = 0 for cos(θ)!=0

Take the inverse cosine of both sides:

θ = 2 π n_1 + π/3 for n_1 element Z or θ = 2 π n_2 + (5 π)/3 for n_2 element Z

or sin(θ) = 0 for cos(θ)!=0

Take the inverse sine of both sides:

θ = 2 π n_1 + π/3 for n_1 element Z

or θ = 2 π n_2 + (5 π)/3 for n_2 element Z

or θ = π n_3 for cos(θ)!=0 and n_3 element Z

The roots θ = π n_3 never violate cos(θ)!=0, which means this assumption can be omitted:

Answer: θ = 2 π n_1 + π/3 for n_1 element Z

or θ = 2 π n_2 + (5 π)/3 for n_2 element Z

or θ = π n_3 for n_3 element Z

Solve for θ over the integers:

tan(θ) - 2 sin(θ) = 0

Factor sin(θ) from the left-hand side:

sin(θ) (sec(θ) - 2) = 0

Split sin(θ) (sec(θ) - 2) into separate parts with additional assumptions.

Assume cos(θ)!=0 from sec(θ):

sec(θ) - 2 = 0 or sin(θ) = 0 for cos(θ)!=0

Add 2 to both sides:

sec(θ) = 2 or sin(θ) = 0 for cos(θ)!=0

Take the reciprocal of both sides:

cos(θ) = 1/2 or sin(θ) = 0 for cos(θ)!=0

Take the inverse cosine of both sides:

θ = 2 π n_1 + π/3 for θ element Z and n_1 element Z or θ = 2 π n_2 + (5 π)/3 for θ element Z and n_2 element Z

or sin(θ) = 0 for cos(θ)!=0

The roots θ = 2 π n_1 + π/3 violate θ element Z for all n_1 element Z:

θ = 2 π n_2 + (5 π)/3 for θ element Z and n_2 element Z

or sin(θ) = 0 for cos(θ)!=0

The roots θ = 2 π n_2 + (5 π)/3 violate θ element Z for all n_2 element Z:

sin(θ) = 0 for cos(θ)!=0

Take the inverse sine of both sides:

Answer: θ = π n_3 for θ element Z and cos(θ)!=0 and n_3 element Z

assume that a and b are positive integers. for each statement below, determine whether the statement is true or false, and indicate your answer in the appropriate box

Answers

Given Statement (a!)b = ab! is false becuase it is not appropriate to represent factorial like this.

What is Factorial?

In mathematics, the term "Factorial" refers to the sum of all positive integers that are less than or equal to a certain positive integer, which is followed by an exclamation point. Thus, factorial seven is denoted by the symbol 7!, which stands for 1*2*3*4*5*6*7. Factorial 0 is equivalent to 1 by definition.

(a!)*b = ab!

this depends on the value of a and b.

I'm not sure if the phrase "ab!" means to multiply a by b and then compute the factorial of the resulting product, as in (ab)!, or to compute the factorial of b first and then multiply this by a, as in a(b!). Either way, the assertion is false.

Given Question is incomplete Complete Question here:

assume that a and b are positive integers. for each statement below, determine whether the statement is true or false, and indicate your answer in the appropriate box  (a!)b = ab!  

To know more about factorial here:

https://brainly.com/question/25505833

#SPJ4

How much would you need to deposit in an account now in order to have $5,000.00 in the account in 821 days?

Assume the account earns 5 % simple interest.

You would need to deposit ___
in your account now.

Answers

The principal investment required to get an accrued amount of $5,000.00 in 821 days is  $4,494.52.

What is the principal need for the investment?

Simple interest is expressed as;

A = P(1 + rt)

Where A is accrued amount, P is principal, r is interest rate and t is time.

Given that;

Accrued amount A = $5,000.00Elapsed time t = 821 days = 821/365 yrsInterest rate r = 5% = 5/100 = 0.05Principal P = ?

Plug the given values into the above formula and solve for principal P.

A = P(1 + rt)

P = A / (1 + rt)

P = $5,000.00 / ( 1 + ( 0.05 × 821/365 ) )

P = $5,000.00 / ( 1 + 821/7300 )

P = $5,000.00 / ( 8121/7300 )

P = $4,494.52

Therefore, the principal required is $4,494.52.

Learn more about simple interest here: brainly.com/question/25845758

#SPJ1

If the VO2 max test scores were normally distributed with a mean, median, and mode of 44ml/kg/min and standard deviation of 8, which of the following would be true. a.We should expect 16% of college students to score at least 36 on the VO2max test. b. We should expect 50% of college students to score 44 on the VO2max test. c. We should expect 84% of college students to score at least 36 on the VO2max test. d. We should expect at least 68% college students to score 36 on the VO2max test.

Answers

The VO2max test resulted by z score should result in a minimum score of 36 for 84% of college students.

To find the answer, we will use the z-score formula:

z = (x - μ)/σ

We will use the given values for the mean (μ = 44), the standard deviation (σ = 8), and the value we are looking for (x = 36).

z = (36 - 44) / 8

z = -1

We can then look up the percentage of values equal to or below -1 on the standard normal distribution table. This value is 84%. Therefore, we should expect 84% of college students to score at least 36 on the VO2max test.

Learn more about z score here

https://brainly.com/question/15016913

#SPJ4

all the edges of a cube are expanding at a rate of 4 in. per second. how fast is the volume changing when each edge is 10in. long?

Answers

The rate at which the volume of the cube is changing is 1200 in³/seconds.

What is volume?

Volume is the space occupied by a solid object.

To calculate the rate at which the volume of the cube is changing, we use the formula below.

Formula:

dV/dt = (dL/dt)×(dV/dL)................ Equation 1

Where:

dV/dt = Rate at which the volume of the cube is changing dL/dt = Rate at which the edge of the cube is expandingdV/dL = Change in the volume of the cube with respect to the edge.

From the question,

Given:

dL/dt = 4 in. per secondsL = 10 in

If, the volume of a cube is V = L³,

Then,

dV/dL = 3L²  = (3×10²) = 300 in²

Substitute these values into equation 1

dV/dt = 4×300dV/dt = 1200 in³/seconds

Hence, the rate at which the volume is changing is 1200 in³/seconds.
Learn more about volume here: https://brainly.com/question/1972490

#SPJ1


If I earn 100% more than you, then you must earn 50% less than me

Answers

Step-by-step brake down:
you: {number like 10} more than me
me: needs {half like 5} less than you

if you get 100% I would start with 0 so 0% + 50%

I don't know if I gave you what you need but I tried

Lance Industries borrowed $130,000. The company plans to set up a sinking fund that will repay the loan at the end of 18 years. Assume a 6% interest rate compounded semiannually. What amount must Lance Industries pay into the fund each period? (Round your answer to the nearest cent.)

Answers

The amount must Lance Industries pay into the fund each period is $2,054.49

Define Sinking Funds

A fund set aside to cover a certain future expense is known as a sinking fund. The sinking fund can be thought of as an annuity if equal contributions are made on a regular basis; its value will rise until it is sufficient to pay the future expense.

Given,

Lance Industries borrowed = $130,000

Time = 18 years

Interest rate = 6%

The future value is,

FVₙ =PMT( (1+i/m)ⁿ - 1 ) / (i/m) )

PMT = payment fund semiannually.

i = 6% or 0.06

m = 2  (semiannually will be 2 times per year)

n = 18 * 2 = 36 periods

FV₃₆ = $130,000

Now, plug in the values in formula

FVₙ =PMT( (1+i/m)ⁿ - 1 ) / (i/m) )

130,000 = PMT ( (1 + 0.06/2)³⁶ - 1 ) / (0.06/2) )

130,000 = PMT ( (1.03)³⁶ - 1 ) / (0.03) )

PMT = 130,000 * ( 0.03 / (1.03)³⁶ - 1 ) )

        = $2,054.49

Hence, the amount must Lance Industries pay into the fund each period is $2,054.49.

To read more about Sinking Funds.

https://brainly.com/question/26361248

#SPJ1

find the least integer n such that n! is divisible by[tex]10^{6}[/tex]

Answers

Answer:

25!

in this type of questions you need to find 10 makers ( it's means you need to find numbers which has 2 or 5 in their prime factors) obviously there is a bunch of numbers which has 2 in their prime factors.

so we need to find multiples of 5. But you should be aware that some numbers have more than one prime factors of 5 e.g. 25 .

ok let's go back to the question.

starting from 1 , we've got 5, 10 , 15 ,20 &25(5²)

so the answer is 25!

Function A is represented by the equation y= 2x+1. Function B is a linear function that goes through the points shown in the table. X 1 3 4 6 3 11 15 23 Which statement correctly compares the rates of change of the two functions?

A. the rate of change of function a is 1
The rat of change of function b is 4

B. the rate of change of function a is 2
the rate of change of function b is 8

C. the rate of change of function a is 1
the rate of change of function b is 8

D. the rate of change of function a is 2
the rate of change of function b is 4

Answers

The rate of change of the linear function A is 2, and the one of function B is 4, then the correct option is D.

Which statement correctly compares the rates of change?

A general linear function is written as:

y = a*x + b

Where a is the rate of change and b is the y-intercept.

In this case, we know that function A is:

y = 2x + 1

Then the rate of change is 2, and the y-intecept is 1.

We also know that if a line passes through two points (x₁, y₁) and (x₂, y₂), then the rate of change of that line is given by the formula:

a = (y₂ - y₁)/(x₂ - x₁)

In the case of function B we have the table:

x: 1   3  4  6

y: 3   11  15  23

We can use any two points on the table, if we use the first two ones:

(1, 3) and (3, 11) we will get:

a = (11 - 3)/(3 - 1) = 8/2 = 4

The rate of change of function B is 4, then the correct statement is the one in option D.

Learn more about linear functions:

https://brainly.com/question/4074386

#SPJ1

john is jogging on a straight jogging trail toward a flagpole at a speed of 500 ft/min. he is currentyl 1500 feet from the flagpole.

Answers

As per the unitary method, it take 3 minutes will it take John to reach the flagpole.

The term unitary method in math is defined as in which the value of one article is first obtained to find out the value of any required articles.

Here we have given that John is jogging on a straight jogging trail toward a flagpole at a speed of 500 ft/min. He is currently 1500 feet from the flagpole. And here we have to write an absolute value function that models John's distance from the flagpole after x minutes.

Here let us consider x be the time taken by John to reach the flagpole.

And here we know that

speed = 500 ft/min

height = 1500 feet

Then the absolute function for the given situation is written as,

=> f(x) = 500|x|

Apply the value of f(x) as 1500, then we get

=>  1500 = 500|x|

=> x = 3

To know more about Unitary method here.

https://brainly.com/question/28276953

#SPJ4

When you are doing a survey of the time it takes for a person to run 100m, which unit would you use?

Answers

When you are doing a survey of the time it takes for a person to run 100m, the unit that you would you use is seconds.

What is unit?

A unit of measurement is a definite magnitude of a quantity that is defined and adopted by convention or law and is used as a standard for measuring the same type of quantity. Any other such quantity can be expressed as a multiple of the unit of measurement. A length, for example, is a physical quantity.

A unit is any standard that is used to compare measurements. Unit conversions allow measurements of a property to be converted from one unit to another, such as centimeters to inches.

In this case, time is measured in seconds.

Learn more about unit on:

https://brainly.com/question/28464

#SPJ1


I need help with this please help me

Answers

The translation rule that maps A(-4, -2), B(-2, 6) and C (4, 4) to A' (-2, -2) B' (0, -10) and C' (6, -8) is

(x + 2, y + 4)

How to find he translation rule that suit the transformation

The transformation rule is solved by comparing the both preimage and image transformations as follows

preimage  image

A(-4, -2)   A' (-2, -2)

B(-2, 6)     B' (0, -10)

C (4, 4)     C' (6, -8)

x direction

-4 + x = -2, x = -2 + 4 = 2

-2 + x = 0, x = 0 + 2 = 2

4 + x = 6, x = 6 - 4 = 2

y direction reflection on the x axis leads to change of sign

preimage  image

A(-4, -2)   A' (-2, 2)

B(-2, 6)     B' (0, 10)

C (4, 4)     C' (6, 8)

-2 + y = 2, y = 2 + 2 = 4

6 + y = 10, y = 10 - 6 = 4

4 + y = 8, y = 8 - 4 = 4

The transformation rule is (x + 2, y + 4)

Learn more about transformation here:

https://brainly.com/question/29321312

#SPJ1

Suppose that survey is planned to estimate the proportion of population that is left-handed The sample data will be used to form confidence interval: Which one of the following combinations of sample size and confidence leve will give the widest interval? n E 500, confidence level 909 n = 1,000, confidence level 909 n = 500, confidence level 959 n = 1,000, confidence level 959 Explain why this sample size and confidence level will give the widest interval: The Select-- sample size and Select- confidence level will both tend to increase the width of the confidence interval:

Answers

The 500 sample size and 95% confidence level will both tend to increase the width of the confidence interval.

In this question, we have been given  the combinations of sample size and confidence level.

We need to select a combinations of sample size and confidence level that will give the widest interval.

We know that the confidence level is typically set in the range of 99% to 80%. The 95% confidence interval will be wider than the 90% interval and which will be wider than the 80% interval.

As we know increasing the confidence will increase the margin of error which results in a wider interval.

But increasing the sample size decreases the width of confidence intervals, as it decreases the standard error.

This means, for the widest interval we need to select a high confidence interval and small sample size.

Therefore, the sample size = 500 and the confidence interval = 95% will give the widest interval.

Learn more about the confidence interval here:

https://brainly.com/question/22851322

#SPJ4

find the indefinite integral using integration by parts with the given choices of u and dv. x2 ln(x) dx;

Answers

Indefinite integral using integration by parts from x2 ln(x) dx is

[tex]\frac{1}{3}[/tex] [tex]X^{3\\}[/tex] (ln(x) - [tex]\frac{1}{3}[/tex]) + C

The given choice of u and dv

[tex]x^{2} ln(x) dx[/tex]

we can use the formula

∫ u dv = uv - ∫ v du

Then split the component

u =  [tex]ln(x)[/tex]     du = [tex]\frac{1}{x}[/tex][tex]dx[/tex]

dv = [tex]x^{2}[/tex]         v = [tex]\frac{1}{3}x^{3}[/tex]

∫ u dv = uv - ∫ v du

[tex]x^{2} ln(x) dx[/tex]   =  ln(x) [tex]\frac{1}{3}x^{3}[/tex] - ∫ [tex]\frac{1}{3}x^{3}[/tex] [tex]\frac{1}{x}[/tex] dx

                    = [tex]\frac{1}{3}x^{3}[/tex] ln(x) - [tex]\frac{1}{3}[/tex] [tex]x^{3}[/tex] [tex]\frac{1}{x}[/tex] dx

                    = [tex]\frac{1}{3}x^{3}[/tex] ln(x) - [tex]\frac{1}{3}[/tex][tex]x^{3} . x^{-1}[/tex] dx

                    = [tex]\frac{1}{3}x^{3}[/tex] ln(x) - [tex]\frac{1}{3}[/tex][tex]x^{2}[/tex] dx     ----> [tex]x^{2}[/tex] dx = [tex]\frac{1}{3} x^{3} + C[/tex]

                    =  [tex]\frac{1}{3}x^{3}[/tex] ln(x) - [tex]\frac{1}{3}[/tex] [tex]\frac{1}{3} x^{3} + C[/tex]

                    = [tex]\frac{1}{3} x^{3} ( ln(x) - \frac{1}{3} ) + C[/tex]

Therefore Indefinite integral using integration by parts

[tex]x^{2} ln(x) dx[/tex]  is  [tex]\frac{1}{3} x^{3} ( ln(x) - \frac{1}{3} ) + C[/tex]

More about integration by parts on this link

https://brainly.com/question/1550271

#SPJ4

mathematics and statistics teacher rex boggs, from australia, weighed the bar of soap in his shower stall before showering in the morning. the data on day and soap weight (in grams) appear in the table. (notice that he forgot to weigh the soap on some days!) here are the data.

Answers

As per the concept of linear regression, the equation of the data is y ​= 133.1807 − 6.3096x.

The term linear regression means to determine the character and strength of the association between a dependent variable and a series of other independent variables.

Here we have given that mathematics and statistics teacher rex boggs, from Australia, weighed the bar of soap in his shower stall before showering in the morning. And the data on day and soap weight (in grams) appear in the table.

Here we need to find the linear regression equation that represents the given situation.

Based on the concept of linear regression, we know the the general formula for the regression is written as,

=> y = a + bx

FRom the given data, we have identified the value of the functions as,

a = 133.1807

b = -6.3096

r² = 0.9962

r = -0.9981

When we apply these values on the formula, then we get the equation as,

Then it implies the regression line is,

=> ŷ = a + bx = 133.18076.3096x

here a represents the day and y represents the weight in grams.

To know more about regression here

https://brainly.com/question/14184702

#SPJ4


Find the sum of following two quantities if the number is reprensented by x
i. Seven multiplied by the sum of nine and a number.
ii. The sum of four and the number.

Answers

The sum of two (63 + 7x) and (4 + x) quantities will be 8x + 67.

What is Algebra?

Algebra is the study of abstract symbols, while logic is the manipulation of all those ideas.

The acronym PEMDAS stands for Parenthesis, Exponent, Multiplication, Division, Addition, and Subtraction. This approach is used to answer the problem correctly and completely.

Let 'x' be the number.

The first number is given as,

⇒ 7(9 + x)

⇒ 63 + 7x

The second number is given as,

⇒ 4 + x

Then the sum of two numbers is given as,

⇒ 63 + 7x + 4 + x

⇒ 8x + 67

The sum of two (63 + 7x) and (4 + x) quantities will be 8x + 67.

More about the Algebra link is given below.

https://brainly.com/question/953809

#SPJ1

The revenue and cost functions of a company are given as () = −5 2 + 750 and () = 100 + 20,000 respectively. (i) Determine the maximum revenue for the company (4 marks) (ii) By plotting the graph of () and () on the same axes, determine the break-even levels. (

Answers

The maximum revenue of the problem given at the end of the answer is of: $144.

$28,125.

The break-even point is of:

(50, 25000) and (80, 28000).

How to maximize revenue?

The revenue function is given as follows:

R(x) = -5x² + 750x.

Which is a quadratic equation with the coefficients given as follows:

a = -5, b = 750.

Then the x-coordinate of the vertex is of:

x = -b/2a = -750/-10 = 75.

Thus the maximum revenue is of:

R(12) = -5(75)² + 750(75) = $28,125.

What are the break-even points?

The break-even points are the points of intersection of the revenue and of the cost functions, given as follows:

Revenue: R(x) = -5x² + 750x.Cost: 100x + 20000.

From the intersection of the graph given at the end of the answer, these points are of:

(50, 25000) and (80, 28000).

More can be learned about maximizing revenue at https://brainly.com/question/13799721

#SPJ1

Other Questions
why was there conflict between native born wasp americans and irish catholic immigrants in big cities Oranges cost $3.08 for 4 pounds . How much does 1 pound cost ? PLEASE HELP! 30 POINTS!!!Please draw or show a car moving at a constant speed which of the following pairs of substances would you expect to form homogeneous solutions when combined? (a) CCl4 and H2O (b) KCl and H2O (c) Br2 and (d)CCl4 _______ run through the thalamus to the primary somatosensory cortex.a.Chemoreceptorsb. Olfactory receptorsc. Thermoreceptors What are some benefits of representing data sets using frequency distributions What are some benefits of using graphs of frequency distributions? What are some benefits of representing data sets using frequency distributions? a. Organizing the data into frequency distribution can make patterns within the data more evident: b. Organizing the data into frequency distribution makes it possible to graph quantitative datac. It is easier t0 determine the minimum and maximum values of a data set when has been arranged into a frequency distribution. What are some benefits of using graphs of frequency distributions? a. It can be easier to determine the class boundaries by looking at a graph of the frequency distribution_ b. Graphing a frequency distribution makes possible to determine the relative frequencies of each of the classes_ c. It can be easier to identify patterns of a data set by looking at a graph of the frequency distribution_ d. Graphing frequency distribution makes it possible to find the total number of observations. which of the following initializes an array of 4 doubles called temperatures with the following initial values: 80.2, 65.3, 12.17, 20.21? economic change country experienced increases in prices of over 1,000% at the beginning of its transition allows private enterprise, but only on a very limited basis started transitioning in the late 1970s which statement is true about cultural norms that govern nonverbal expressiveness?A. American emoticons focus on eye expressions, whereas Japanese emoticons feature the mouth. B. The nationality or ethnicity of a person is far more important than the culture in which they live. C. The facial expressions of Japanese nationals are highly similar to those of Japanese Americans. D. Overt demonstrations of happiness are not equally encouraged in all cultures. in addition to recording current costs, the also provides valuable information when bidding on similar jobs in the future. (enter only one word per blank.) What mass of calcium hydroxide is formed when 10. 0 g of calcium oxide reacts with 10. 0 g of water?. well, the astronaut finally wakes up because he hears two distinct alarms. one for high co2 and another for low pressure, but the suit has way too much oxygen. as he stumbles to the hab, he is facing both high co2 and high o2, but a lowered atmospheric pressure. if the atmospheric pressure inside the suit is now 0.5 atm (equivalent to an altitude on earth of 5500 m above sea level), then what low pressure symptoms could he be experiencing? this is separate from just the co2 and o2 levels. g which of the following antibiotics is recommended for use against gram-negative bacteria? a) polyenes b) bacitracin c) cephalosporin d) penicillin e) polymyxin please help i need it for my examCaptain John Smith wrote books based on his journals to persuade people to colonize America. How would a story like his capture by Powhatan and rescue by Pocahontas appeal to people who were considering moving to colonial America? Which of the following is an example of the barter system?AMatthew wants to exchange his clothes with his twin brother, but he doesn't wish to exchange.BGia pays ninety rupees to a barber for a hair cut.CAnurag wants ten packet of chips in exchange for the one chocolate bar that he has, but his friend does not wish the same.DMatthew agress to give Anurag his clothes and is happy to receive the chocolate in order to analyze, descriptive information must go through a process called . question 22 options: 1) codes. 2) content analysis. 3) codes analysis. 4) none of the above. Balance the reaction of H2 + O2 = H2O using this chemical equation balancer! natural selection alters the likelihood of an organism passing on its genes to subsequent generations. the evolutionary effects of natural selection are only apparent when looking at an entire population over time. select the reasons why evolutionary effects of natural selection are only apparent when looking at populations. selection changes the natural variance of phenotypes. during their lifetime, individuals become adapted to their environments. the accumulation of beneficial alleles requires multiple generations. phenotypes with an underlying genetic component are inherited. only beneficial traits are passed on to the next generation. based on your chapter, a full understanding of sexuality would need to include erotic desires, beliefs, and behaviors, as well as a camarera is a _____.hostessseatdetectivewaitress